4
$\begingroup$

Is there a finite set of primes, $S \subset Spec(\mathbb{Z})$ such that if $K$ over $\mathbb{Q}$ is a number field such that every $p \in S$ is completely split in $K$ then $K=\mathbb{Q}$? If so, what do we know about such $S$'s?

$\endgroup$
3
  • 1
    $\begingroup$ I don't think such S exists. If you can find an odd prime q such that all the primes in S are congruent to 1 mod q then every prime in S is totally split in q-cyclotomic field. So, my example boils down to: Given a finite set S of primes is it always possible to find a prime q as above? maybe some analytic number theorist knows the answer of this of the top of his/ger head. $\endgroup$ Jan 8, 2011 at 22:49
  • 6
    $\begingroup$ However there will be an infinite, density zero set of primes with this property. Because you can just enumerate the non-trivial extensions of the rationals as $K_1$, $K_2,\ldots$ and then for each $n$ choose a prime $p>10^{10^n}$ which isn't completely split in $K_n$ and add it to $S$. The ridiculous growth rate I assumed forces the set to be very sparse. $\endgroup$ Jan 8, 2011 at 23:04
  • 1
    $\begingroup$ @Guillermo: no. Take {2, 3}. $\endgroup$ Jan 8, 2011 at 23:59

3 Answers 3

15
$\begingroup$

This is not even true for quadratic extensions. Given primes $p_1, p_2, ... p_n$ find a prime $q \equiv 1 \bmod 4p_1 ... p_n$, which exists by Dirichlet's theorem, and consider $K = \mathbb{Q}(\sqrt{q})$.

$\endgroup$
1
  • 1
    $\begingroup$ For this application it should be possible to manage without even citing Dirichlet: construct an algebraic integer $\alpha$ in an appropriate polyquadratic extension such that $\alpha$ is not a unit and every factor of the norm of $\alpha$ must be split. (This is a natural generalization of the elementary proof that there are infinitely many primes congruent to $1 \bmod 4$, or to $1 \bmod 3$.) $\endgroup$ Nov 29, 2019 at 1:44
7
$\begingroup$

No. In fact, given a finite set of primes $S=\lbrace p_1,p_2,\ldots,p_n\rbrace$, we can find a finite extension $K$ of $\mathbb{Q}$ in which elements of $S$ decompose in any way that we like. That is, for any numbers $r_i\ge1$ and $e_{i1},e_{i2},\ldots,e_{ir_i}\ge1$, we can find an extension $K$ in which $p_i$ decomposes as $$ p_i\mathcal{O}_K=\mathfrak{p}_{i1}^{e_{i1}}\mathfrak{p}_{i2}^{e_{i2}}\cdots\mathfrak{p}_{ir_i}^{e_{ir_i}} $$ for all $i=1,\ldots,n$, with $\mathfrak{p}_{ij}$ being distinct primes of $\mathcal{O}_K$. Furthermore, if $f_{ij}$ are positive integers with $d=\sum_{j=1}^{r_i}e_{ij}f_{ij}$ independent of $i$ then this can always be done such that $[K\colon\mathbb{Q}]=d$ and $[\mathcal{O}_K/\mathfrak{p}_{ij}\colon\mathbb{Z}/p_i\mathbb{Z}]=f_{ij}$. For $p_i$ to split completely, this just means that we use $e_{ij}=f_{ij}=1$ and $r_i=d$.

Here's a construction I came up with: Let $k_{ij}$ be an extension of the $p_i$-adic numbers $\mathbb{Q}_{p_i}$ in which $p_i$ has ramification index $e_{ij}$ and such that the extension of residue fields is of degree $f_i$. We can do this by adding an $e_{ij}$'th root of $p_i$ to the splitting field of $X^{p_i^{f_i}}-X$ over $\mathbb{Q}_{p_i}$. Letting $R_{ij}$ be the elements of $k_{ij}$ which are integral over $\mathbb{Z}_{p_i}$, the primitive element theorem implies that $k_{ij}=\mathbb{Q}_{p_i}(x_{ij})$ for some $x_{ij}\in R_{ij}$. Let $g_{ij}\in\mathbb{Z}_{p_i}[X]$ be the minimal (monic) polynomial of $x_{ij}$. This will have degree $[k_{ij}\colon \mathbb{Q}_{p_i}]=e_{ij}f_{ij}$. Also, $g_{ij}$ will be irreducible in $\mathbb{Z}/p_i^r\mathbb{Z}$ for sufficiently large $r$. Set $g_i=\prod_{j}g_{ij}$ which is monic of degree $d$.

Now, using the chinese remainder theorem, there will be a monic degree $d$ polynomial $g\in\mathbb{Z}[X]$ such that $g=g_i$ mod $p_i^r$ for each $i$, and $f=X^d+p$ mod $p^2$, for some prime $p\not\in S$. Eisenstein's criterion implies that $g$ is irreducible in $\mathbb{Q}[X]$, and Hensel's lemma implies that in $\mathbb{Q}_{p_i}[X]$, $g$ splits into a product of irreducible terms equal to $g_{ij}$ mod $p_i^r$. Taking $K=\mathbb{Q}(x)$ for a root $x$ of $g$ gives the claimed extension.

$\endgroup$
1
  • $\begingroup$ This is a great answer emphasizing many number-theory results. Thanks! $\endgroup$ Aug 31, 2014 at 2:21
5
$\begingroup$

No. If d is a square modulo p for all p in S, then all p in S split in $\mathbb{Q}(\sqrt d)$.

$\endgroup$

Your Answer

By clicking “Post Your Answer”, you agree to our terms of service and acknowledge you have read our privacy policy.

Not the answer you're looking for? Browse other questions tagged or ask your own question.